Difference between revisions of "1987 AJHSME Problems/Problem 3"

(New page: ==Problem== <math>2(81+83+85+87+89+91+93+95+97+99)=</math> <math>\text{(A)}\ 1600 \qquad \text{(B)}\ 1650 \qquad \text{(C)}\ 1700 \qquad \text{(D)}\ 1750 \qquad \text{(E)}\ 1800</math> ...)
 
m (Problem: CHOICES Syntax)
 
(20 intermediate revisions by 8 users not shown)
Line 5: Line 5:
 
<math>\text{(A)}\ 1600 \qquad \text{(B)}\ 1650 \qquad \text{(C)}\ 1700 \qquad \text{(D)}\ 1750 \qquad \text{(E)}\ 1800</math>
 
<math>\text{(A)}\ 1600 \qquad \text{(B)}\ 1650 \qquad \text{(C)}\ 1700 \qquad \text{(D)}\ 1750 \qquad \text{(E)}\ 1800</math>
  
==Solution==
+
==Solution 1==
 +
Find that
 +
<cmath>(81+83+85+87+89+91+93+95+97+99) = 5 \cdot 180</cmath>
 +
Which gives us
 +
<cmath>\begin{align*}
 +
2(5 \cdot 180) &= 10 \cdot 180\\
 +
&= 1800 & \text{ Thus \boxed{\text{E}} is the correct answer}
 +
\end{align*}</cmath>
  
{{Solution}}
+
==Solution 2==
 +
<math>2(81+83+85+87+89+91+93+95+97+99)</math>
 +
Pair the least with the greatest, second least with the second greatest, etc, until you have five pairs, each adding up to <math>81+99</math> = <math>83+97</math> = <math>85+95</math> = <math>87+93</math> = <math>89+91</math> = <math>180</math>.  Since we have <math>5</math> pairs, we multiply <math>180</math> by <math>5</math> to get <math>900</math>.  But since we have to multiply by 2 (remember the 2 at the beginning of the parentheses!), we get <math>1800</math>, which is <math>\boxed{\text{E}}</math>.
  
 
==See Also==
 
==See Also==
 
+
{{AJHSME box|year=1987|num-b=2|num-a=4}}
[[1987 AJHSME Problems]]
+
[[Category:Introductory Algebra Problems]]
 +
{{MAA Notice}}

Latest revision as of 01:03, 22 January 2020

Problem

$2(81+83+85+87+89+91+93+95+97+99)=$

$\text{(A)}\ 1600 \qquad \text{(B)}\ 1650 \qquad \text{(C)}\ 1700 \qquad \text{(D)}\ 1750 \qquad \text{(E)}\ 1800$

Solution 1

Find that \[(81+83+85+87+89+91+93+95+97+99) = 5 \cdot 180\] Which gives us \begin{align*} 2(5 \cdot 180) &= 10 \cdot 180\\ &= 1800 & \text{ Thus \boxed{\text{E}} is the correct answer} \end{align*}

Solution 2

$2(81+83+85+87+89+91+93+95+97+99)$ Pair the least with the greatest, second least with the second greatest, etc, until you have five pairs, each adding up to $81+99$ = $83+97$ = $85+95$ = $87+93$ = $89+91$ = $180$. Since we have $5$ pairs, we multiply $180$ by $5$ to get $900$. But since we have to multiply by 2 (remember the 2 at the beginning of the parentheses!), we get $1800$, which is $\boxed{\text{E}}$.

See Also

1987 AJHSME (ProblemsAnswer KeyResources)
Preceded by
Problem 2
Followed by
Problem 4
1 2 3 4 5 6 7 8 9 10 11 12 13 14 15 16 17 18 19 20 21 22 23 24 25
All AJHSME/AMC 8 Problems and Solutions

The problems on this page are copyrighted by the Mathematical Association of America's American Mathematics Competitions. AMC logo.png